1
$\begingroup$

Given a finite (multi)set of elements $\{x_1, \ldots, x_n\}$ the arithmetic mean $\mathsf{AM}$ is less than or equal to the maximum element call it $\max$. In otherwords, $\mathsf{AM} \leq \max$. But equality only happens when all elements in the population are equal. Another way to put this is when the average absolute deviation from the mean is $0$. Call the average absolute deviation from the mean $\mathsf{AAD}$. I would like to extend the $\mathsf{AM} \leq \max$ to include $\mathsf{AAD}$ as well.

To do so, I considered an extreme case: Every element but $1$ is $\max$ and one element has a minimal deviation (call it $\mathsf{minD}$). Thus, the average of this extreme case establishes the following inequality: $$\mathsf{AM} \leq \frac{\max + \max + \ldots + (\max - \mathsf{minD})}{n}$$ $$\mathsf{AM} \leq \frac{n\max}{n} - \frac{\mathsf{minD}}{n}$$ $$\mathsf{AM} \leq \max - \frac{\mathsf{minD}}{n}$$

I wanted to relate this back to $\mathsf{AAD}$ so I observed that $\mathsf{AAD}$ is also an average of a population, thus, by Samuelson's Inequality we have:

$$\mathsf{minD} \geq \mathsf{AAD} - \text{stddev}(\mathsf{AAD})\sqrt{n-1}$$

subbing back in we get

$$\mathsf{AM} \leq \max - \frac{\mathsf{AAD} - \text{stddev}(\mathsf{AAD})\sqrt{n-1}}{n}$$

Using inequality (3) from here we get an upperbound for $\text{stddev}(\mathsf{AAD})$ of

$$\text{stddev}(\mathsf{AAD}) \leq \sqrt{(\max - \mathsf{AAD})(\mathsf{AAD} - \min)}$$

Plugging back in we get:

$$\mathsf{AM} \leq \max - \frac{\mathsf{AAD} - \sqrt{(\max - \mathsf{AAD})(\mathsf{AAD} - \min)(n-1)}}{n}$$

But I would like to know if this bound can be improved and if the $\min$ term can be removed. Any guidance including hints would be appreciated.

$\endgroup$

0

You must log in to answer this question.

Start asking to get answers

Find the answer to your question by asking.

Ask question

Explore related questions

See similar questions with these tags.